What is the Use of the Constant in the Integral?

  • Thread starter Thread starter songoku
  • Start date Start date
Click For Summary
The discussion centers on the use of constants in integrals, specifically in defining the function f(x) as an integral from p to x of cos(t)/t dt. Participants clarify that if p is set to 3, the integral evaluates to 0 at x=3, raising the question of what constant should be added to the integral to satisfy f(3) = 4. There is also a correction regarding the derivative, with emphasis on ensuring f'(x) equals cos(x)/x. The conversation highlights the importance of properly defining the limits and constants in integral calculus. Understanding these concepts is crucial for accurately solving related problems.
songoku
Messages
2,508
Reaction score
402
Homework Statement
Find f(x) where ##f(x)=\frac{\cos x}{x}## and f(3) = 4. State the answer in form of ##f(x)=\int_{t=p}^{t=q} (........)##
Relevant Equations
Fundamental Theorem of Calculus
This is my attempt:
$$f(x)=\int_{t=p}^{t=x} \frac{\cos t}{t} dt$$

But I am not sure what ##p## is and what the use of ##f(3)=4##

Thanks
 
Physics news on Phys.org
You forgot about the constant that is added to the integral. If you start the integral at p=3, then you know that the integral part is 0 at x=3. So what constant is added to the integral?
 
  • Like
Likes songoku and Delta2
songoku said:
Homework Statement:: Find f(x) where ##f(x)=\frac{\cos x}{x}## and f(3) = 4. State the answer in form of ##f(x)=\int_{t=p}^{t=q} (...)##
Did you forget to add the prime? Shouldn't it be ##f'(x) = \frac{\cos x}{x}##?
 
  • Like
Likes songoku, Delta2 and berkeman
FactChecker said:
You forgot about the constant that is added to the integral. If you start the integral at p=3, then you know that the integral part is 0 at x=3. So what constant is added to the integral?
I understand

Mark44 said:
Did you forget to add the prime? Shouldn't it be ##f'(x) = \frac{\cos x}{x}##?
Yes, I am sorry

Thank you very much FactChecker and Mark44
 
Question: A clock's minute hand has length 4 and its hour hand has length 3. What is the distance between the tips at the moment when it is increasing most rapidly?(Putnam Exam Question) Answer: Making assumption that both the hands moves at constant angular velocities, the answer is ## \sqrt{7} .## But don't you think this assumption is somewhat doubtful and wrong?

Similar threads

  • · Replies 3 ·
Replies
3
Views
2K
Replies
1
Views
2K
Replies
6
Views
2K
Replies
9
Views
2K
  • · Replies 23 ·
Replies
23
Views
2K
  • · Replies 11 ·
Replies
11
Views
3K
Replies
1
Views
1K
Replies
12
Views
2K
  • · Replies 9 ·
Replies
9
Views
1K
  • · Replies 105 ·
4
Replies
105
Views
6K